Difference between revisions of "2005 AIME II Problems/Problem 11"
Ragnarok23 (talk | contribs) m (→See Also) |
|||
Line 1: | Line 1: | ||
== Problem == | == Problem == | ||
− | + | Let <math> m </math> be a positive integer, and let <math> a_0, a_1,\ldots,a_m </math> be a sequence of integers such that <math> a_0 = 37, a_1 = 72, a_m = 0, </math> and <math> a_{k+1} = a_{k-1} - \frac 3{a_k} </math> for <math> k = 1,2,\ldots, m-1. </math> Find <math> m. </math> | |
+ | |||
== Solution == | == Solution == | ||
+ | |||
== See Also == | == See Also == | ||
*[[2005 AIME II Problems]] | *[[2005 AIME II Problems]] |
Revision as of 22:30, 8 July 2006
Problem
Let be a positive integer, and let be a sequence of integers such that and for Find